Question and Answers Forum

All Questions      Topic List

Logarithms Questions

Previous in All Question      Next in All Question      

Previous in Logarithms      Next in Logarithms      

Question Number 89702 by 974342176 last updated on 18/Apr/20

3^(2t+1) +3^(t+2) =((10)/3)

$$\mathrm{3}^{\mathrm{2}{t}+\mathrm{1}} +\mathrm{3}^{{t}+\mathrm{2}} =\frac{\mathrm{10}}{\mathrm{3}} \\ $$

Answered by Kunal12588 last updated on 18/Apr/20

3^t =p  3p^2 +9p=10/3  9p^2 +27p−10=0  9p^2 +30p−3p−10=0  (3p−1)(3p+10)=0  p=(1/3);p=((−10)/3)  3^t =(1/3)⇒t=−1  3^t =((−10)/3)⇒no sol^n

$$\mathrm{3}^{{t}} ={p} \\ $$$$\mathrm{3}{p}^{\mathrm{2}} +\mathrm{9}{p}=\mathrm{10}/\mathrm{3} \\ $$$$\mathrm{9}{p}^{\mathrm{2}} +\mathrm{27}{p}−\mathrm{10}=\mathrm{0} \\ $$$$\mathrm{9}{p}^{\mathrm{2}} +\mathrm{30}{p}−\mathrm{3}{p}−\mathrm{10}=\mathrm{0} \\ $$$$\left(\mathrm{3}{p}−\mathrm{1}\right)\left(\mathrm{3}{p}+\mathrm{10}\right)=\mathrm{0} \\ $$$${p}=\frac{\mathrm{1}}{\mathrm{3}};{p}=\frac{−\mathrm{10}}{\mathrm{3}} \\ $$$$\mathrm{3}^{{t}} =\frac{\mathrm{1}}{\mathrm{3}}\Rightarrow{t}=−\mathrm{1} \\ $$$$\mathrm{3}^{{t}} =\frac{−\mathrm{10}}{\mathrm{3}}\Rightarrow{no}\:{sol}^{{n}} \\ $$

Commented by MJS last updated on 18/Apr/20

right path wrong conclusion  3^t =p ⇔ t=((ln p)/(ln 3))  p=−((10)/3)∨p=(1/3)  ⇒  t=((ln −((10)/3))/(ln 3)) ∉R∨ t=((ln (1/3))/(ln 3))=−1  • solution: t=−1 •  (t=((ln ((10)/3))/(ln 3))+((2n+1)/(ln 3))πi ∈C)

$$\mathrm{right}\:\mathrm{path}\:\mathrm{wrong}\:\mathrm{conclusion} \\ $$$$\mathrm{3}^{{t}} ={p}\:\Leftrightarrow\:{t}=\frac{\mathrm{ln}\:{p}}{\mathrm{ln}\:\mathrm{3}} \\ $$$${p}=−\frac{\mathrm{10}}{\mathrm{3}}\vee{p}=\frac{\mathrm{1}}{\mathrm{3}} \\ $$$$\Rightarrow \\ $$$${t}=\frac{\mathrm{ln}\:−\frac{\mathrm{10}}{\mathrm{3}}}{\mathrm{ln}\:\mathrm{3}}\:\notin\mathbb{R}\vee\:{t}=\frac{\mathrm{ln}\:\frac{\mathrm{1}}{\mathrm{3}}}{\mathrm{ln}\:\mathrm{3}}=−\mathrm{1} \\ $$$$\bullet\:\mathrm{solution}:\:{t}=−\mathrm{1}\:\bullet \\ $$$$\left({t}=\frac{\mathrm{ln}\:\frac{\mathrm{10}}{\mathrm{3}}}{\mathrm{ln}\:\mathrm{3}}+\frac{\mathrm{2}{n}+\mathrm{1}}{\mathrm{ln}\:\mathrm{3}}\pi\mathrm{i}\:\in\mathbb{C}\right) \\ $$

Terms of Service

Privacy Policy

Contact: info@tinkutara.com